G x H: Proving Cyclicity with GCD = 1

  • Thread starter Thread starter hsong9
  • Start date Start date
  • Tags Tags
    Gcd
Click For Summary
To prove that the direct product G x H of cyclic groups G and H is cyclic when gcd(m, n) = 1, one must show that an element a = (g, h) generates the entire group. Since the least common multiple of the orders of g and h is equal to mn, the group G x H has mn elements. It is essential to ensure that g and h are generators of their respective groups, as not all elements have the same order as the group itself. The hint provided suggests demonstrating that the order of (g, h) is indeed mn, confirming that G x H is cyclic. Therefore, the proof is valid if it includes the clarification that at least one g in G must have order equal to |G| for G to be cyclic.
hsong9
Messages
71
Reaction score
1

Homework Statement


Let G and H be cyclic groups, with |G| = m and |H| = n. If gcd(m,n) =1, show that G x H is cyclic.


The Attempt at a Solution


Let a = (g,h) in G x H. Then |a| = lcm (|g|,|h|).
Since gcd(m,n)=1, then lcm (m,n) = mn.
Thus lcm (|g|,|h|) = lcm (m,n) = mn.
so <a> = G x H has mn elements and a cyclic group.
Right?
 
Physics news on Phys.org
Basically ok. But you want to be a little more careful with your choice of g and h. It's not true that for every element g in G that |g|=|G|. E.g. |e|=1. And G could have nontrivial subgroups. You'd better make sure that g and h are generators.
 
so.. you mean I have to make sense that for every element g in G that |g|=|G|?
If so..
Actually, the problem has Hint that [ G=<g> and H = <h>, show |(g,h)| = mn].
If consider this hint, my answer is ok?
or any other thing needs to prove?
 
Last edited:
The point is that at least one g in G satisfies |g|=|G|. Otherwise, would it be a cyclic group? That's what the proof is missing.
 
Question: A clock's minute hand has length 4 and its hour hand has length 3. What is the distance between the tips at the moment when it is increasing most rapidly?(Putnam Exam Question) Answer: Making assumption that both the hands moves at constant angular velocities, the answer is ## \sqrt{7} .## But don't you think this assumption is somewhat doubtful and wrong?

Similar threads

  • · Replies 3 ·
Replies
3
Views
3K
  • · Replies 8 ·
Replies
8
Views
4K
  • · Replies 9 ·
Replies
9
Views
2K
  • · Replies 1 ·
Replies
1
Views
2K
  • · Replies 2 ·
Replies
2
Views
2K
  • · Replies 1 ·
Replies
1
Views
2K
  • · Replies 5 ·
Replies
5
Views
2K
  • · Replies 1 ·
Replies
1
Views
2K
  • · Replies 1 ·
Replies
1
Views
2K
  • · Replies 3 ·
Replies
3
Views
3K